You are on page 1of 40

M3C3

LEGALEDGE TEST SERIES


Part of the Most Awesome and Consistently Successful Study Material and Test Series Module, spanning across
both Physical and Online Programs in the entire Country. While most of the world fumbled and faltered, 2020
has been another inspiring Success story both for us and those who chose to trust us. As a result LE was able to
engineer Clean-Sweep-Landslide figures of a handsome 35 Selections under 100 ranks, and a whopping 180
selections under 500 ranks in CLAT 2020. With AILET being no different, a total of 30 LEtians found their way
into NLUD in 2020. Read on!

MOCK COMMON LAW ADMISSION TEST 2022-23


MOCK CLAT #03

Candidate Name : _________________


Duration : 120 Minutes
Batch : _________________
Max. Marks : 150
Contact No. : _________________
Centre Name : __________
Date of Exam : _________________

INSTRUCTIONS TO CANDIDATES

1. No clarification on the question paper can be sought. Answer the questions as they are.
2. There are 150multiple choice objective type questions.
3. Each question carries ONE mark. Total marks are 150.
4. There is a negative marking of 0.25 marks for every incorrect answer.
5. Candidates have to indicate the correct answer by darkening one of the four responses provided, with a
BALL PEN (BLUE OR BLACK) in the OMR Answer Sheet.
Example: For the question, "Where is the TajMahal located?", the correct answer is (b).
The candidate has to darken the corresponding circle as indicated below :
(a) Kolkata (b) Agra (c) Bhopal (d) Delhi
Right Method Wrong Methods

6. Answering the questions by any method other than the method indicated above shall be considered
incorrect and no marks will be awarded for the same.
7. More than one response to a question shall be counted as wrong.
8. The candidate shall not write anything on the OMR Answer Sheet other than the details required and in
the spaces provided for.
9. After the Test is over, the candidate has to return the OMR Answer Sheet to the invigilator. The
candidate should take the Test Paper along with them.
10. The use of any unfair means by any candidate shall result in the cancellation of his/her candidature.
11. Impersonation is an offence and the candidate, apart from disqualification, may have to face criminal
prosecution.
12. Electronic gadgets like mobile phones, pagers or calculators are strictly not permitted inside the Test
Centre/Hall.
13. The candidates shall not leave the hall before the Test is over.
SECTION-A : ENGLISH LANGUAGE

Directions (Q.1-Q.28): Read the passages carefully and answer the questions.

Passage (Q.1-Q.5): Read the passage carefully and answer the questions that follow.
No one would have believed, in the last years of the nineteenth century, that human affairs were being
watched keenly and closely by intelligences greater than man’s and yet as mortal as his own; that as men
busied themselves about their affairs they were scrutinized and studied, perhaps almost as narrowly as a
man with a microscope might scrutinize the transient creatures that swarm and multiply in a drop of
water. With infinite complacency men went to and fro over this globe about their little affairs, serene in
their assurance of their empire over matter. It is possible that the infusoria under the microscope do the
same. No one gave a thought to the older worlds of space as sources of human danger, or thought of
them only to dismiss the idea of life upon them as impossible or improbable. It is curious to recall some of
the mental habits of those departed days. At most, terrestrial men fancied there might be other men upon
Mars, perhaps inferior to themselves and ready to welcome a missionary enterprise. Yet, across the gulf
of space, minds that are to our minds as ours are to those of the beasts that perish, intellects vast and
cool and unsympathetic, regarded this earth with envious eyes, and slowly and surely drew their plans
against us. And early at the twentieth century came the great disillusionment.
The planet Mars, I scarcely need remind the reader, revolves about the sun at a mean distance of
140,000,000 miles, and the light and heat it receives from the sun is barely half of that received by this
world. It must be, if the nebular hypothesis has any truth, older than our world, and long before this earth
ceased to be molten life upon its surface must have begun its course. The fact that it is scarcely one-
seventh of the volume of the earth must have accelerated its cooling to the temperature at which life
could begin. It has air and water, and all that is necessary for the support of animated existence.
Yet so vain is man, and so blinded by his vanity, that no writer, up to the very end of the nineteenth
century, expressed any idea that intelligent life might have developed there far, or indeed at all, beyond
its earthly level. Nor was it generally understood that since Mars is older than our earth, with scarcely a
quarter of the superficial area, and remoter from the sun, it necessarily follows that it is not only more
distant from life’s beginning but nearer its end.
[Excerpts from 'The War of The Worlds' by H.G.Wells]

1. Which of the following option represent the meaning of the word ‘infusoria’ as has been used in the
passage?
(a) Small aquatic rocks (b) A branch of philosophy
(c) A mental state of anger (d) Minute aquatic creatures

2. Which of the following statement/s is/are correct according to the passage?


Statement 1: The Earth receives twice the amount of heat and light from Sun as compared to Mars.
Statement 2: The Earth is seventeen times bigger than Mars.
(a) Statement 1 (b) Statement 2
(c) Neither statement 1 nor 2 (d) Both

3. The main theme of the passage revolves around:


(a) Man’s vanity and denial about the existence of extra-terrestrial life forms of same or superior
intelligence.
(b) Disproving the existence of any other life forms other than humans through scientific logics.
(c) Criticizing the authors who write about extra terrestrial life forms and undermine human intelligence.
(d) None of the above.

Head Office: 127, Zone II, MP Nagar, Bhopal |+91-7676564400| https://www.toprankers.com Page 2 of 40
4. ‘No one would have believed, in the last years of the nineteenth century, that human affairs were being
watched keenly and closely by intelligences greater than man’s and yet as mortal as his own; that as men
busied themselves about their affairs they were scrutinized and studied, perhaps almost as narrowly as a
man with a microscope might scrutinize the transient creatures that swarm and multiply in a drop of
water.’
Which of the following literary devices has been used in the italicized sentence above?
(a) Foreshadowing, Metaphor (b) Cliffhanger, Allusion
(c) Dramatic Irony, Humor (d) Allegory, Motif

5. ‘And early at the twentieth century came the great disillusionment.’


Spot the error in the underlined sentence above.
(a) error in preposition (b) error in article
(c) error in conjunction (d) error in verb

Passage (Q.6-Q.9): Read the following passage carefully and answer the questions that follow.
It’s hard to explain the ways in which Farah Bashir’s book Rumours of Spring feels special. It’s an
engaging new memoir by a woman who has grown up amid war, bloodshed and those tumultuous
years of Kashmir that consumed so many lives and tore so many families apart.
Bashir writes from the perspective of an adolescent girl who is appalled at the suffering – both
material and emotional – that has visited her people. Even though Bashir is raised in a relatively well-
off family in a Srinagar neighbourhood, she is not just a mere spectator of conflict which has ravaged
Kashmir for 30 years. She has experienced it. She is its victim.
This past fortnight, former governor of J&K Jagmohan Malhotra, who would call the shots in the Valley
during the most volatile months of early 1990s, died. His demise provoked a flurry of commentary.
And his book, My Frozen Turbulence, was once again in the public spotlight. Like every other
narrative set by state functionaries, Malhotra’s book also conjures the same projections: people
crazed into fanaticism by their religious beliefs, militancy as unprovoked aggression against the state,
good Kashmiris who are naturally Indian versus bad Kashmiris who buy into Pakistani propaganda.
These characterizations have since crystallized and become enduring motifs at mainstream
Indian discourse. Until, of course, Kashmiris decided to write. Bashir’s book strikes the reader with
remarkable freshness, like a draught of cold wind blowing against a sweaty face.
Her story is not her alone but of dozens of people she has been associated with and whose lives were
enmeshed in the political conflict and turned upside down. It includes her grandmother, Bobeh, who
appears to be the moving spirit behind the book, and whose asthma is worsened by repeated
exposure to acrid teargas exhausts; Rajj Mas, a professional funeral bather whose son is abducted by
the Indian forces, and has never shown up since; Vaseem, who is Bashir’s first love interest and to
whom she would write letters via mail until the post office in Srinagar catches fire. In a turmoil-ridden
Kashmir Valley, no one restores the charred and dysfunctional mail box as a result of which they both
lose contact.

6. The main theme of the passage revolves around:


(a) Portraying a picture of religious fanaticism in Kashmir as the plot for Bashir’s book.
(b) Narrating a plot comparing good Kashmiris and bad Kashmiris.
(c) Narrating the review of an autobiographical book with several other characters enmeshed in political
conflicts in Kashmir.
(d) Comparing the book of Bashir with the book by the former Governor of J&K Jagmohan Malhotra.

7. Which of the following statement/s is/are true according to the passage?


Statement 1: Bashir’s love interest was a professional funeral bather’s son who went missing after
abducted by the Indian forces.
Statement 2: Repeated exposure to teargas affected Bashir’s grandmother’s asthma.
(a) Statement 1 (b) Statement 2
(c) Neither Statement 1 nor 2 (d) Both

Head Office: 127, Zone II, MP Nagar, Bhopal |+91-7676564400| https://www.toprankers.com Page 3 of 40
8. Which of the following literary styles has been used by the author in the above passage?
(a) Narrative (b) Expository (c) Descriptive (d) Persuasive

9. ‘Bashir’s book strikes the reader with remarkable freshness, like a draught of cold wind blowing against a
sweaty face.’
Which of the following literary device has been used in the italicized line above?
(a) Irony (b) Metaphor (c) Anamorphism (d) Simile

Passage (Q.10-Q.13): Read the following passage carefully and answer the questions that follow.

Unlike other games, championships and competitions of this world, the game of life is not seasonal but
one that is on all the time. It is a game between our soul and the mind. The soul is trying to reach its goal,
and the mind is focused on keeping the soul from it. The mind does so through the mechanism of
thoughts — thoughts of the past and worries of the future — all of which are a means to distract the
soul. We experience this mechanism of the mind whenever we take steps toward the soul’s goal.
When we sit down to meditate or attempt to help someone in need, the mind tries to convince us we
should be spending our time in other ways. How then can the soul win this game of life? To win, our soul
needs to experience God’s love. While the mind tries to make us love the objects of this world,
saints advise us to open the horizon of our limited vision and realise that there is an existence
beyond the physical world.
They tell us there are regions within that occur concurrently with this physical world — regions that will
enlighten us, give us peace, help us experience the truth and put us in harmony with the Divine. They
teach us the technique of meditation through which we can experience these regions.
Once we do so, the inner riches remain with us, and the mind is unable to cast doubt. Connection with
God’s love nourishes and strengthens our soul, enabling it to triumph over the mind and reach its goal.

10. The main theme of the passage revolves around :


(a) Explaining the reasons to perform supernatural practices to create physical harmony.
(b) Explaining the drawbacks of organized religion.
(c) Explaining the importance and ways of harmonizing one’s soul and mind.
(d) Explaining the disadvantages of atheism.

11. Which of the following statements is/are false according to the passage?
Statement 1: The purpose of the soul is to distract the mind through worries of the future.
Statement 2: We spontaneously experience a positive mechanism of our mind to encourage the soul to
reach its goal.
(a) Statement 1 (b) Statement 2
(c) Neither Statement 1 nor 2 (d) Both

12. Which of the following writing style has been followed by the author in the above passage?
(a) Narrative (b) Expository (c) Persuasive (d) Descriptive

13. While mind tries to make us love objects of this world, saints advise us to open horizon of our
limited vision and realise that there is existence beyond physical world.
Find the option that represents the correct order of missing articles in the italicized sentence above.
(a) the, the, a, an (b) the, the, the, an, the
(c) the, a, the, an (d) the, the, the, the

Head Office: 127, Zone II, MP Nagar, Bhopal |+91-7676564400| https://www.toprankers.com Page 4 of 40
Passage (Q.14-Q.18): Read the following passage carefully to answer the questions that follow.
Scarlett is worried about changing school after the summer. She worries that she's too short for her age
and that the other children at the school will make fun of her. She shows me some pictures of the school
she is at now and her classmates. I look at the picture and it shows children of all heights and shapes
and sizes. Some are tall, some are short, some are fat and some are thin. Some are black and some are
white, and most of them are somewhere in between. Some have red hair and some have blond hair,
some have long hair and some have short hair.
I tell her not to worry about the new school, tell her that she'll be OK, and ask her about the new subjects
she'll be studying. She tell me that she's worried about learning French, and I tell her not to worry, that it
isn't a very difficult language. She tells me that she already knows five languages.
'Five languages!' I shout. 'That's impossible! How do you already know five languages?'
'Because I've got five languages in my body,' she says.
I ask her what she means, and she starts to tell me the story of her family. Some of the story I already
know. I've already heard stories about her grandfather. He was from Scotland; he was a sailor, but not a
very good sailor, so he only got as far as Portsmouth, a big navy town on the south coast of England, not
very far from Scotland at all. When he got to Portsmouth, he stopped there, left the navy and became a
boxer. He lost fights and drank a lot. However, he still managed to see the world by meeting a woman
who came from Laos. Nobody really knows how this woman had ended up in Portsmouth, but she still
lives there, and I tell Scarlett that she should try and find out her grandmother's story. 'No, she's too old
now,' says Scarlett, 'and anyway, she's lived in Portsmouth nearly all her life.'
Scarlett's grandparents were only together long enough to produce a son – probably one of the only
Scottish-Laotians in the world. They called him Bill, which is usually short for 'William', but his name was
just 'Bill'. Bill inherited his father's personality and his mother's looks, so the only thing he thought he
could do was become a rock star. He never really managed to become a rock star, though, so now he
works as a graphic designer.

14. The main theme of the passage revolves around:


(a) Narrating the experience of the protagonist growing up in a multi-cultural household.
(b) Explaining the importance of being a multilingual.
(c) Describing the Protagonist through narrating accounts of her family history.
(d) Criticising the protagonist’s family for being not supportive.

15. Which of the following statements/s is/are true according to the passage?
Statement 1: Sailing from Scotland to the South coast of England doesn’t require great sailing skills.
Statement 2: Scarlett’s grandparents had a long marriage full of love and happiness.
(a) Statement 1 (b) Statement 2
(c) Neither statement 1 nor 2 (d) Both

16. 'Because I've got five languages in my body,' she says.


Which of the following literary technique has been used in the above sentence in context to the passage?
(a) Foreshadowing (b) Extended Metaphor
(c) Imagery (d) Oxymoron

17. Which of the following statement/s is/are false according to the passage?
Statement 1: Scarlett’s grandmother was from Portsmouth.
Statement 2: Bill is Scarlett’s cousin brother who works as a Graphic Designer.
(a) Statemnet 1 (b) Statement 2
(c) Statement 1 and 2 both (d) Neither statement 1 nor 2

Head Office: 127, Zone II, MP Nagar, Bhopal |+91-7676564400| https://www.toprankers.com Page 5 of 40
18. She tell me that she's worried about learning French, and I tell her not to worry, that it isn't a very difficult
language.
Spot the error in the above bold and underlined sentence.
(a) Error in verb (b) Error in punctuation
(c) Error in conjunction (d) Error in preposition

Passage (Q.19-Q.23): Read the passage carefully and answer the questions that follow.
A Child was standing on a street-corner. He leaned with one shoulder against a high board-fence and
swayed the other to and fro, the while kicking carelessly at the gravel.
Sunshine beat upon the cobbles, and a lazy summer wind raised yellow dust which trailed in clouds down
the avenue. Clattering trucks moved with indistinctness through it. The child stood dreamily gazing.
After a time, a little dark-brown dog came trotting with an intent air down the sidewalk. A short rope was
dragging from his neck. Occasionally he trod upon the end of it and stumbled.
He stopped opposite the child, and the two regarded each other. The dog hesitated for a moment, but
presently he made some little advances with his tail. The child put out his hand and called him. In an
apologetic manner the dog came close, and the two had an interchange of friendly pattings and waggles.
The dog became more enthusiastic with each moment of the interview, until with his gleeful caperings he
threatened to overturn the child. Whereupon the child lifted his hand and struck the dog a blow upon the
head.
This thing seemed to overpower and astonish the little dark-brown dog, and wounded him to the heart.
He sank down in despair at the child's feet. When the blow was repeated, together with an admonition in
childish sentences, he turned over upon his back, and held his paws in a peculiar manner. At the same
time with his ears and his eyes he offered a small prayer to the child.

He looked so comical on his back, and holding his paws peculiarly, that the child was greatly amused and
gave him little taps repeatedly, to keep him so. But the little dark-brown dog took this chastisement in the
most serious way, and no doubt considered that he had committed some grave crime, for he wriggled
contritely and showed his repentance in every way that was in his power. He pleaded with the child and
petitioned him, and offered more prayers.
At last the child grew weary of this amusement and turns toward home. The dog was praying at the time.
He lay on his back and turned his eyes upon the retreating form. Presently he struggled to his feet and
started after the child. The latter wandered in a perfunctory way toward his home, stopping at times to
investigate various matters. During one of these pauses he discovered the little dark-brown dog who was
following him with the air of a footpad.

19. Which of the following writing styles has been followed by the author in the passage?
(a) Narrative (b) Descriptive (c) Persuasive (d) Expository

20. Which of the following statement/s is/are true according to the passage?
Statement 1: The child beat up the dog because it was creating nuisance for him.
Statement 2: It was a rainy day when the story takes place.
(a) Statement 1 (b) Statement 2
(c) Neither statement 1 nor 2 (d) Both

21. At last the child grew weary of this amusement and turns toward home.
Which of the following errors are present in the underlined sentence above?
(a) Error in verb (b) Error in punctuation
(c) Error in adverb (d) Error in pronoun

Head Office: 127, Zone II, MP Nagar, Bhopal |+91-7676564400| https://www.toprankers.com Page 6 of 40
22. Which of the following statement/s is/are true?
Statement 1: The dog became very fond of the child within a very short span of time.
Statement 2: The child depicted in this passage is of an agile nature.
(a) Statement 1 (b) Statement 2
(c) Neither statement 1 nor 2 (d) Both

23. The latter wandered in a perfunctory way toward his home, stopping at times to investigate various
matters.
Which of the following options is meant by ‘ the latter’ in this sentence?
(a) The Dog (b) The child
(c) The person accompanying the child (d) The owner of the dog.

Passage (Q.24-Q.28): Read the passage carefully and answer the questions that follow.
Day had broken cold and grey, exceedingly cold and grey, when the man turned aside from the main
Yukon trail and climbed the high earth- bank, where a dim and little-travelled trail led eastward through
the fat spruce timberland. It was steep bank, and he paused for breath at top, excusing act to himself by
looking at his watch. It was nine o'clock. There was no sun nor hint of sun, though there was not a cloud
in the sky. It was a clear day, and yet there seemed an intangible pall over the face of things, a subtle
gloom that made the day dark, and that was due to the absence of sun. This fact did not worry the man.
He was used to the lack of sun. It had been days since he had seen the sun, and he knew that a few
more days must pass before that cheerful orb, due south, would just peep above the sky- line and dip
immediately from view.
The man flung a look back along the way he had come. The Yukon lay a mile wide and hidden under
three feet of ice. On top of this ice were as many feet of snow. It was all pure white, rolling in gentle
undulations where the ice-jams of the freeze-up had formed. North and south, as far as his eye could
see, it was unbroken white, save for a dark hair-line that curved and twisted from around the spruce-
covered island to the south, and that curved and twisted away into the north, where it disappeared behind
another spruce-covered island. This dark hair-line was the trail--the main trail--that led south five hundred
miles to the Chilcoot Pass, Dyea, and salt water; and that led north seventy miles to Dawson, and still on
to the north a thousand miles to Nulato, and finally to St. Michael on Bering Sea, a thousand miles and
half a thousand more.
But all this--the mysterious, far-reaching hairline trail, the absence of sun from the sky, the tremendous
cold, and the strangeness and weirdness of it all--made no impression on the man. It was not because he
was long used to it. He was a new-comer in the land, a chechaquo, and this was his first winter. The
trouble with him was that he was without imagination. He was quick and alert in the things of life, but only
in the things, and not in the significances. Fifty degrees below zero meant eighty odd degrees of frost.
Such fact impressed him as being cold and uncomfortable, and that was all.

24. Which of the following statement/s is/are true according to the passage?
Statement 1: It was a cold winter night when the protagonist was travelling.
Statement 2: The Yukon is a small local hill.
(a) Statement 1 (b) Statement 2
(c) neither statement 1 nor 2 (d) Both

25. The protagonist of the passage has been depicted as a man who has:
(a) Less imagination hence, courageous
(b) Fell short on intelligence
(c) Lack of optimism but has a good survival strategy.
(d) None of the above

Head Office: 127, Zone II, MP Nagar, Bhopal |+91-7676564400| https://www.toprankers.com Page 7 of 40
26. It was steep bank, and he paused for breath at top, excusing act to himself by looking at his watch.
Spot the error in the underlined sentence above.
(a) Error in article (b) Error in preposition
(c) Error in conjunction (d) Error in noun

27. Which of the following statement/s is/are false according to the passage?
Statement 1: The distance between Nulato and St. Michael’s is five hundred miles.
Statement 2: Not many people travelled towards the east of the Yukon.
(a) Statement 1 (b) Statement 2
(c) Neither statement 1 nor 2 (d) Both

28. It had been days since he had seen the sun, and he knew that a few more days must pass before that
cheerful orb, due south, would just peep above the sky- line and dip immediately from view.
Which of the following literary device has been used in the above italicized sentence?
(a) Verisimilitude (b) Irony (c) Foeshadowing (d) Metaphor

Head Office: 127, Zone II, MP Nagar, Bhopal |+91-7676564400| https://www.toprankers.com Page 8 of 40
SECTION-B : GENERAL KNOWLEDGE/CURRENT AFFAIRS

Directions (Q.29–Q.63): Read the information given below and answer the questions based on it.

Passage (Q.29-Q.33): Union Minister Nitin Gadkari, accompanied by Chief Minister Jai Ram Thakur, on
Thursday visited Atal Tunnel at Rohtang on the Leh-Manali Highway. The [1] tunnel, considered as an
engineering marvel, was dedicated to the nation by Prime Minister on October 3, last year. It connects
Manali to the Lahaul Spiti valley.
The Union Minister and Chief Minister were accorded traditional welcome on entering the north portal
towards Lahaul side by the people of Lahaul Spiti district. The Union Minister while interacting with media
at the south portal, appreciated the efforts of the BRO for constructing this tunnel. He said that as many
as 19 tunnels were being constructed in Himachal Pradesh by the Union Government, out of which work
was in progress on eight tunnels. It was Mr Gadkari's first visit to Himachal after commissioning of this
project. Notably, two more tunnels are likely to come up in the future on the Manali-Leh highway which
holds strategic importance.
Source: Extracted with edits and revisions from the article “Gadkari visits Atal tunnel in Himachal's
Rohtang” by United News of India.
29. Which of the following has been redacted by [1]?
(a) 8.76 km
(b) 9.02 km
(c) 10.04 km
(d) 11.23 km

30. Which of the following is correct?


(a) It is the country’s first tunnel to have an escape tunnel within the main tunnel
(b) It is the longest highway tunnel in the world above the height of 10,000 metres.
(c) It shortens the distance between Manali to Lahaul by 36 km
(d) All of the above

31. This tunnel shall secure almost an all-weather connectivity but not all of it. To ensure full all-weather
connectivity a few more tunnels need to be built. Which of the following is not one of these tunnels?
(a) Tunnel to negotiate the Baralacha Pass
(b) Tunnel to negotiate the Lachung La Pass
(c) Tunnel to negotiate the Tanglang La Pass
(d) Tunnel to negotiate the Jelep La Pass

32. Atal Tunnel has been designed for traffic density of how many cars per day and trucks per day with max
speed of 80 km/hr respectively?
(a) 4000 cars and 2000 trucks
(b) 3000 cars and 1000 trucks
(c) 3000 cars and 1500 trucks
(d) 3500 cars and 1000 trucks

33. Which of the following services are available in the tunnel?


1. Telephone
2. Advertising screens
3. Air quality monitors
4. Fire hydrant
5. Safety-nets
(a) Only 2 and 4
(b) Only 1, 3 and 4
(c) Only 1, 3 and 5
(d) 1, 2, 3, 4 and 5

Head Office: 127, Zone II, MP Nagar, Bhopal |+91-7676564400| https://www.toprankers.com Page 9 of 40
Passage (Q.34-Q.38): The army in Myanmar's second city, Mandalay, has clashed with a local militia
opposed to February's military coup. The clashes are the first time the [1] have come up against the army
in a major city. The defence force is a collective name for militia groups that have sprung up in Myanmar
since the coup. Myanmar's military junta has violently put down anti-coup protests in the country, killing
hundreds of civilians. Until now, actions by the [1] have been limited to fighting in rural areas or small
towns. Tuesday's clashes brought the violence to a major city, marking a new phase in the fallout from
the coup. According to local reports, troops raided a boarding school being used as a base by the militia,
leading to an exchange of gunfire.
The junta said four protesters were killed in the raid and eight arrested, and several soldiers injured. The
Mandalay [1] confirmed the raid, on 54th Street in the Chan Mya Thar Si Township, in a post on social
media. The [1] said weapons were seized from inside the building and several members of the group
arrested, but did not confirm whether there were any casualties.
Source: Extracted with edits and revisions from the article “Myanmar army clashes with anti-junta militia
in major city” of BBC.

34. Which of the following has been redacted by [1]?


(a) Myanmar Democratic Forces (MDF)
(b) Myanmar Defense Forces (MDF)
(c) People’s Democratic Forces (PDF)
(d) People's Defense Forces (PDF)

35. What has been India’s stand in the whole controversy?


(a) As a form of protest, India has cut-off relations with the military junta of Myanmar
(b) India has surprisingly supported and recognized the military junta of Myanmar
(c) India has offered to mediate between the two parties
(d) India has not cut-off relations with the Myanmar military

36. Which of the following ports is being jointly developed by India and Myanmar?
(a) Thilawa Port
(b) Dawei Port
(c) Sittwe Port
(d) Yangon Port

37. What is the name of the party that is backed by the Military in Myanmar?
(a) Union Solidarity and Development Party (USDP)
(b) National League for Democracy (NLD)
(c) Union Solidarity and Democratic Party (USDP)
(d) National League for Development (NLD)

38. Who is the Myanmar Military Chief?


(a) Win Myint
(b) Min Aung Hlaing
(c) Than Shwe
(d) Kyu Kyu Hla

Head Office: 127, Zone II, MP Nagar, Bhopal |+91-7676564400| https://www.toprankers.com Page 10 of 40
Passage (Q.39-Q.45): The Parliamentary Standing Committee on Science and Technology,
Environment, Forests and Climate Change has recommended that the government assuage concerns
raised over the DNA Technology (Use and Application) Regulation Bill, 2019, including over creation of a
national databank of crime scene DNA profiles and fears of communities being targeted. While
recognising the importance of DNA technology in criminal investigation, the committee, in its report tabled
in Parliament Wednesday, says, “The risk with a national databank of crime scene DNA profiles is that it
will likely include virtually everyone since DNA is left at the ‘crime scene’ before and after the crime by
several persons who may have nothing to do with the crime being investigated.”
It adds, “These fears (regarding the Bill) are not entirely unfounded (and) have to be recognized and
addressed by the government and by Parliament as well… The Committee is of the strong opinion that
an enabling ecosystem must be created soon to ensure that DNA profiling is done in a manner that is
fully consistent with the letter and spirit of various Supreme Court judgments and with the Constitution.”

39. In which year was this Bill first recommended?


(a) 2001
(b) 2002
(c) 2003
(d) 2004

40. Which of the following are important issues that have been raised by the Parliamentary panel?
(a) Violation of Right to Privacy
(b) Inadequate Supporting Infrastructure
(c) Both a and b
(d) Neither a nor b

41. Who is the Chairman of this committee?


(a) Asaduddin Owaisi
(b) Jairam Ramesh
(c) Shashi Tharoor
(d) Tejasvi Surya

42. Which of the following apprehensions been raised by the committee?


1. There can be misuse in caste-based profiling
2. It can be used for political gains
3. It has inadequate supporting structure.
(a) Only 1 and 2
(b) Only 2 and 3
(c) Only 1 and 3
(d) 1, 2 and 3

43. Which of the following instances can arise as a result of this Bill?
1. State-sanctioned biological surveillance may be allowed
2. There can be instances of planting of evidence
(a) Absence of a Data Protection Law
(b) There can be instances of planting of evidences
(c) Both a and b
(d) Neither a nor b

44. Which of the following Constitutional articles are in danger to being violated by the Bill?
(a) Article 19(2)
(b) Article 20(2)
(c) Article 20(3)
(d) All of the above
Head Office: 127, Zone II, MP Nagar, Bhopal |+91-7676564400| https://www.toprankers.com Page 11 of 40
45. How many types of persons does the Bill seek to establish an index of?
(a) 3
(b) 4
(c) 5
(d) 6

Passage (Q.46-Q.51): The Global Hunger Index 2020 report released on Friday has ranked India at 94
among [1] countries. India was ranked [2] out of 117 countries in the Global Hunger Index 2019 that was
published jointly by Concern Worldwide and Welthungerhilfe.
In 2018, India had ranked 103 among 119 countries on the Global Hunger Index.
The peer-reviewed annual report aims to measure and track hunger at the global, regional and country
levels. In its latest report, GHI puts worldwide hunger at a "moderate" level while warning that nearly 690
million people across the world are undernourished.
With a score of 27.2 out of 50, the 2020 Global Hunger Index report terms the level of hunger in India as
"[3]".
It is important to note that India's score on the GHI index in 2020 (27.2) is an improvement as compared
to 38.9 in 2000, 37.5 in 2006, and 29.3 in 2012.
"Data from 1991 through 2014 for Bangladesh, India, Nepal, and Pakistan showed that stunting is
concentrated among children from households facing multiple forms of deprivation, including poor dietary
diversity, low levels of maternal education, and household poverty,” the report has said.

46. Which of the following has been redacted by [1]?


(a) 102
(b) 105
(c) 107
(d) 111

47. Which of the following has been redacted by [2]?


(a) 98
(b) 100
(c) 102
(d) 110

48. Which of the following has been redacted by [3]?


(a) Serious
(b) Low
(c) Moderate
(d) Alarming

49. Which of the following SDG is related to Zero Hunger by 2030?


(a) SDG 2
(b) SDG 12
(c) SDG 13
(d) SDG 14

50. Mark the correct statement regarding GHI 2020.


(a) In terms of stunting rate South Asia’s performance have been degraded since 2000
(b) India experienced an increase in under-five mortality between 2000 and 2018
(c) At present, India lags behind Nepal, Pakistan, Bangladesh, and Indonesia among others on the
Global Hunger Index.
(d) Only 28 out of 107 countries, including Rwanda, Afghanistan, Liberia, and Chad among others have
fared worse than India.

Head Office: 127, Zone II, MP Nagar, Bhopal |+91-7676564400| https://www.toprankers.com Page 12 of 40
51. _______ have been classified as having the highest GHI scores in the world.
(a) Africa South of Sahara
(b) Congo
(c) South East Asian Countries
(d) Central Asia

Passage (Q.52-Q.57): Gulf leaders signed a “[1]” agreement towards ending the diplomatic rift with Qatar
at a summit in Saudi Arabia on Tuesday.
The announcement was made at the Gulf Arab leaders’ meeting in Saudi Arabia, with the annual summit
taking place amid a breakthrough in the dispute between a Saudi Arabia-led bloc and Qatar that started
in June 2017.
Leaders of the six-member Gulf Cooperation Council (GCC) signed the Al-Ula declaration, named after
the Saudi city where the summit is being held, and a final communique.
This is the English translation of the closing statement of the summit in full:
In response to the gracious invitation from the Custodian of the Two Holy Mosques King Salman bin
Abdul Aziz Al Saud, king of the Kingdom of Saudi Arabia, may God protect him, the Supreme Council
held its 41st summit in the city of Al-Ula on Tuesday, 21st Jumada I, 1442 AH, corresponding to January
5, 2021.
The summit was chaired by His Royal Highness Prince Mohammed bin Salman bin Abdulaziz, crown
prince, deputy prime minister and minister of defence of the Kingdom of Saudi Arabia, and was attended
by:
· His Highness Sheikh Mohammed bin Rashid Al Maktoum, vice president and prime minister of the
United Arab Emirates and ruler of Dubai.

52. Which of the following has been redacted by [1]?


(a) Nuclear Armament
(b) Solidarity and Stability
(c) Boycott Terror Attacks
(d) Sustainable development

53. Which of the following members of GCC bordered by Red Sea?


(a) Kuwait
(b) Qatar
(c) UAE
(d) Saudi Arabia

54. Why in 2017 Saudi Arabia severed ties with Qatar?


(a) Qatar attacked Saudi Arabia through land missiles
(b) Qatar by going out of the league, selling oil on cheaper rates
(c) Qatar was charged to be too close to Iran and backed radical Islamist groups
(d) Qatar was interfering in politics of UAE

55. Approximately how much India is importing crude oil from GCC countries?
(a) 20%
(b) 34%
(c) 54%
(d) 72%

Head Office: 127, Zone II, MP Nagar, Bhopal |+91-7676564400| https://www.toprankers.com Page 13 of 40
56. Which of the following TV News Channels was told by the Coalition countries to be shut down in its
demands to the ?
(a) WION
(b) Al Jazeera
(c) Al Arabiya
(d) CP24

57. Which country act as a mediator in settling peace between GCC and Qatar?
(a) Kuwait
(b) USA
(c) Russia
(d) Egypt

Passage (Q.58-Q.63): After strong opposition from trade unions across the country, Sri Lanka has
unilaterally pulled out of a 2019 agreement with India and [1] for developing the strategic East Container
Terminal (ECT) at the Colombo Port. In 2019, India and Sri Lanka signed a memorandum of
understanding for “co-operation on economic projects”. The development and operation of the container
terminal was one of the projects in the MoU: “A Container Terminal in Colombo Port as a Joint Venture,
which includes Indian investments considering that majority of transshipment in Colombo Port is related
to India. GOSL will announce the award of the contract…by end May 2017”.
The MoU did not mention the Eastern Container Terminal, but India and Sri Lanka had already been in
discussion for its development and operation. Although India and Sri Lanka have seemingly friendly ties
and much cultural affinity and people-to-people contact, the relationship is complex — and the majority
[2] public opinion is layered with the memory of Indian intervention in the ethnic conflict.
Source: Extracted with edits and revisions from the article “The Colombo port setback for India” by The
Indian Express

58. Which of the following has been redacted by [1]?


(a) USA
(b) Australia
(c) Japan
(d) South Korea

59. Which of the following has been redacted by [2]?


(a) Lankan Tamil
(b) Sinhala - Buddhists
(c) Lankan Malayalis
(d) Both a and b

60. How many terminals are there in the Colombo Port?


(a) 3
(b) 4
(c) 5
(d) 6

61. Which of the following statements is incorrect?


(a) After removing India, from the ECT project, Lanka has asked India to develop another part and has
offered 85% stake in it.
(b) The APSEZ has been offered to work on this project from the Indian side.
(c) Both a and b
(d) Neither a nor b

Head Office: 127, Zone II, MP Nagar, Bhopal |+91-7676564400| https://www.toprankers.com Page 14 of 40
62. Which of the following terminals is China developing in Sri Lanka?
(a) ECT
(b) CICT
(c) JCT
(d) SAGT

63. China is also working on hybrid wind and solar energy projects on three Sri Lankan islands. Which of the
following is not one of them?
(a) Delft
(b) Nainativu
(c) Analativu
(d) Kachchativu

Head Office: 127, Zone II, MP Nagar, Bhopal |+91-7676564400| https://www.toprankers.com Page 15 of 40
SECTION – C: LEGAL REASONING

Directions (Q.64 – Q.102): Read the comprehensions carefully and answer the questions based on it.

Passage (Q.64-Q.68): The term foreign judgments means the judgment of a foreign court, and Section
13 of the code provides the criteria for enforcement of such judgement in the Indian Court.
There are three primary sources of law about the enforcement of foreign judgments in India:
Laws passed by the parliament: Section 44A of the Code explains the constitutional principle that the
judgment handed down by a Superior Court of the Reciprocating Territory is implemented in India as
though it were a ruling enacted by the Indian District Courts. That being said, a judgment emerging
from a non-reciprocating territory cannot be immediately mandated in the same way and a fresh case
should be issued for its regulation in which such a judgment is of only evidentiary value.
Bilateral treaties with the reciprocating countries concerning recognition and enforcement of foreign
judgments to which India is a party; and
Judicial precedents: A foreign judgment not emanating from a superior court of a reciprocating
territory cannot be executed in India without the filing of a new suit in which the said judgment has
only evidentiary value.
One of the fundamental requirements of recognition is that a foreign judgment must not be inconclusive
under the Code. According to Section 13 of the Code, a foreign judgment will be inconclusive if it:
is pronounced by a court that was not of competent jurisdiction;
is not given on the merits of the case;
appears to be founded on an incorrect view of international law or a refusal to recognize Indian law
(where applicable);
violates principles of natural justice;
is obtained by fraud; or
Sustains a claim founded on a breach of Indian law.
(Excerpt from Diva Rai, Enforcement of foreign judgments in India, May 6, 2021,
https://blog.ipleaders.in/enforcement-of-foreign-judgments-in-india/ )

64. Rakesh, an Indian National, is a trustee to property of Manoharam, a Srilankan National in India for about
ten years. India and Srilanka have the bilateral treaty between them for the enforcement of judgment
since 2004. Recently, a war started between both the countries and Srilanka has been declared an
enemy State. Rakesh decided to sell the property as the trusteeship agreement stands suspended due to
such circumstance. Manoharan sought the injunction passed by the Superior Court of Srilanka. He filed
an application before the Civil Court in India to enforcement the injunction as Srilanka is the
Reciprocating Territory. Rakesh argued that the decree cannot be enforced Srilanka has been declared
as enemy State. Can Manoharan seek the enforcement the decree for injunction before the Indian Civil
Court?
(a) Yes, because the bilateral treaty was entered into prior to the occurrence of war
(b) Yes, because the trusteeship agreement was signed prior to the occurrence of war
(c) No, because Srilanka has been declared as enemy as all the contract stands suspended
(d) No, Srilanka is not a Reciprocating Territory

65. Mohanlal and Sarita married in Chennai in 2008, in compliance with the Hindu Marriage Act. The couple
last settled in New Zealand together and then the husband moved to Australia. Later, in 2014, he filed a
petition for the dissolution of marriage in Australia and secured an irreversible breakup of marriage
decree by legally meeting the condition of 90 days of residency in Australia. The wife had made it known
that she would not consent to the Australian Court’s jurisdiction. She lodged a criminal lawsuit against the
husband before the Family Court, Chennai for bigamy after he married other woman. Can the Court
enforce the foreign decree assuming the fact the Australia has the Reciprocating Territory in the Gazette
of India?
(a) Yes, the foreign decree is admissible in the Indian Court as Australia is a Reciprocating Territory

Head Office: 127, Zone II, MP Nagar, Bhopal |+91-7676564400| https://www.toprankers.com Page 16 of 40
(b) Yes, because the parties have complied with the conditions prescribed under the Australian law for
seeking the decree for dissolution of marriage
(c) No, because the marriage can be dissolved only as per Hindu law
(d) No, because the marriage took place in Chennai as per the Hindu law, hence, Indian Courts have the
jurisdiction for the dissolution of marriage

66. Aramco, a Saudi Company, entered into a contract on February 17, 2017 with Oil and Natural Gas
Commission (ONGC), a Government of India undertaking, for carrying out structural oil drilling in India.
The contract provided that amount payable to the Aramco on fee or any default etc. Shall be computed in
Riyal and the ONGC shall pay 80% of that amount in Riyal and remaining in Indian Rupee using a fixed
conversion rate. The disputes and differences, which arose between the parties, were referred to
arbitration. The award made on May 28, 2020 directed certain payments to be made in Riyal, but did not
specify the rate of exchange for conversion into Indian Rupee. The Delhi High Court passed a decree on
January 2021 in terms of the award simpliciter without fixing any date for conversion of the Riyal into
Indian rupees. Decide as to which date shall be taken into account to compute the conversion rate?
(a) The date of the entering into contract shall be taken as conclusive date to decide the conversion rate
(b) The date of the award given be the Umpire in arbitration shall be taken as conclusive date to decide
the conversion rate
(c) The date of the decree passed by the Delhi High Court shall be taken as conclusive date to decide
the conversion rate
(d) The decree passed by the arbitration is not conclusive as per Section 13 of the CPC

67. Anurag, an Indian and Swetlana, a Dutch, married in 2017 in Pune under the Hindu Law. Prior to the
marriage, they signed the pre-nuptial agreement in the Netherlands, wherein it was decided that in case
of default by one party, the other party has the right to dissolve the marriage unilaterally. Swetlana was
granted the decree to dissolve the marriage by the Appellate Court of the Netherlands. Anurag filed an
application for the injunction before the Family Court, Pune. Can the Family Court enforce the decree
passed by the Supreme Court of the Netherlands, considering that the Netherlands is not a Reciprocating
Territory?
(a) The Court can enforce the decree because it is valid as per the Netherlands law
(b) The Court can enforce the decree because the parties mutually entered into the agreement
(c) The Court cannot enforce the decree because the agreement is against the public policy of India
(d) The Court cannot enforce the decree as the Family Court Pune has the jurisdiction to decide on the
validity of the marital agreement i.e. the pre-nuptial agreement

68. Aaditya works as an Associate in the Linklaters LLP, a multinational law firm in London. The firm has
claimed 4000 £ have been misappropriated by Aaditya while in the employment of the firm, before the
High Court London. As Aaditya was in India at that time, did not appear at the hearing. The High Court
passed a decree based on the documentary evidence presented before the Court. The firm filed the
enforcement suit in the Delhi High Court against Aaditya. Decide will the firm succeed in the enforcement
of the decree, considering the fact that the UK is the Reciprocating Territory?
(a) Yes, because the decree was passed by the highest Court in London
(b) Yes, because the decree was passed by the Court after perusing appropriate evidence
(c) No, because the decree is not conclusive in nature
(d) No, because the firm had the higher bargaining power

Head Office: 127, Zone II, MP Nagar, Bhopal |+91-7676564400| https://www.toprankers.com Page 17 of 40
Passage (Q.69-Q.73): On 9th April 2021, the Kerala High Court in Treasa Josfine v. State of
Kerala directed the State authorities to consider an application submitted by the petitioner (a female
trainee engineer) for the post of a Safety Officer in Kerala Minerals and Metals Ltd, on the grounds that
she had been denied opportunity on the basis of her sex.
The counter affidavit filed by the State argued that the notification was in compliance with Section
66(1)(b) of the Factories Act, 1948. Section 66(1) (b) states “no woman shall be required or allowed to
work in any factory except between the hours of 6 A.M. and 7 P.M”. The submission of the State was that
the post of a Safety Officer required the person so engaged to work round the clock, even during the
night if required. Therefore, women could not be hired for this position under the provisions of the
Factories Act.
In Treasa Josfine, Justice Anu Sivaraman agreed that Section 66(1) (b) is a beneficial provision intended
to protect women. However, the Court observed that the Factories Act was enacted at a different time
and in a different socio-economic context, particularly with respect to the roles played by women in
society. Given this context, Section 66(1) (b)’s force could only be utilized to protect women, but would
not constitute a reason to deny them engagement and opportunity if they are fully qualified [paragraph
15]. On this basis, the Court set aside the notification and held it to be violative of Articles 14, 15 and 16.
Notably, states such as Maharashtra and Kerala have altered the position under Section 66(1)(b) by
permitting the employment of women post 7 p.m. provided that all safety and security safeguards are met
by the employer.
(Excerpt from Unnati Ghia, Deconstructing the Paternalism in Section 66(1)(b) – Treasa Josfine v. State of Kerala,
Indian Constitutional Law and Philosophy, May1st, 2021
https://indconlawphil.wordpress.com/2021/05/01/guest-post-deconstructing-the-paternalism-in-section-
661b-treasa-josfine-v-state-of-kerala/)

69. Which of the following inference can be drawn be perusing the information given in the passage?
(a) Section 66 (1) (b) is per se in contravention to Article 14, 15 and 16 of the Constitution
(b) The enactment of provisions, such as, Section 66 (1) (b), was very much essential for socio-economic
development of women in 1948
(c) The Court Order leaves the employment of women entirely to the option of the employer, but does
little to incentivize them
(d) The State must identify and attempt to remedy specific forms of disadvantage

70. A statute was passed by the state of Jinjibar in the Union of Mastana that prohibits women from working
for more than 10 hours a day, keeping in view of their performance of maternal functions and household
responsibilities. A group of male employees challenged the Statute on the ground of violation of Article
14, 15 and 16. Considering that the laws of the Union of Mastana are pari materia to the laws of the
Union of India, which of the following could be the most appropriate argument in favour of the Statute?
(a) The Statute would ensure that the State must identify and attempt to remedy specific forms of
disadvantage
(b) The Statute mandate safeguards and security for women at the workplace without removing them
from the workplace altogether
(c) Women have to devote time to the home, because it further entrenches the public-private divide.
(d) Women holding household duties is not a universal phenomenon

Head Office: 127, Zone II, MP Nagar, Bhopal |+91-7676564400| https://www.toprankers.com Page 18 of 40
71. Supposedly, the state government of Punjab has made an amendments in the Section 30 of the Punjab
Excise Act, 1914 that prohibits employment of "any man under the age of 25 years" or "any woman" in
any part of such premises in which liquor or intoxicating drug is consumed by the public. Ramesh, a
member of Hotel Association of India challenged the validity of the said provision on the basis of violative
of Articles 19(1) (g), 14 and 15 of the Constitution of India. Decide the validity of the provision as per the
legal information given in the passage.
(a) The provision is constitutional under the law as it seeks to protect the vulnerable groups in such kinds
of services
(b) The provision is unconstitutional under the law as it does not have reasonable nexus with the
objective of the provision
(c) The provision is unconstitutional under the law as it prohibits the women from seeking employment in
hotel industry
(d) The provision is constitutional under the law as it does have reasonable nexus with the objective of
the provisions

72. In the Union of Shahistan, there is a law i.e. Air Shahistan Employees' Service Regulations; and
Regulation 12 (1) (a) of Air Shahistan Employees' Service Regulations provides that the services of the
Air Hostesses, except the cabin crew members of similar status, would stand terminated on first
pregnancy. Considering the fact that the law of the Union of Shahistan is pari materia to the Union of
India, decide the constitutionality of the Regulation keeping in view the kinds of requirements pertaining
to the services of an airhostess.
(a) The Regulation is constitutional having regard to the nature of job functions, the mode of
recruitments of Air Hostesses, their qualifications, the Air Hostesses fall within a category
separate from the other
(b) Te recruitment of the Air Hostesses is actually sex based recruitment made not on the ground of
sex alone but swayed by a lot of other considerations and hence it is Constitutional in nature
(c) The Regulation is unconstitutional as The impugned provisions appear to be a clear case of official
arbitrariness
(d) The Regulation is unconstitutional as it is a hostile discrimination and not reasonable classification

73. Smt. R. Vasantha, who is employed in a textile mill and is willing to work during day time or at night time
on shift basis, depending on the workload. The Mill owner has informed her that the management will
render dormitories for comfortable stay and food within the factory premises, transport if necessary and
other facilities to enable the women employees to work in the night shifts. However, due to the operation
of Section 66(1) (b) of the Factories Act, 1948, she is unable to carry on her chosen employment
between 7.00 P.M. and 6.00 A.M. and hence she wants to challenge Section 66(1) (b) of the Factories
Act, 1948. Support her with requisite arguments as per the legal information given in the passage.
1. The provision discriminates the factory workers based on sex and gender and such gender
discrimination is arbitrary and without any basis
2. The said piece of legislation is one of the instances of romantic paternalism by which men wanted to
restrict women to household activities so that they can retain their economic superiority in the homes
3. Good intent or absence of discriminatory intent does not immunize an employment practice from the
equal opportunity requirement under Articles 15 and 16 of the Constitution of India
(a) All of the above
(b) Only 1 and 3
(c) Only 1 and 2
(d) Only 2 and 3

Head Office: 127, Zone II, MP Nagar, Bhopal |+91-7676564400| https://www.toprankers.com Page 19 of 40
Passage (Q.74-Q.77): A group administrator cannot be held vicariously liable for an act of a member of
the group, who posts objectionable content, unless it is shown that there was common intention or pre-
arranged plan acting in concert pursuant to such plan by such member of a Whatsapp group and the
administrator. When an online platform is created, the creator thereof cannot expect any of the members
thereof to indulge in defamation and defamatory statements made by any member of the group cannot
make the Administrator liable therefor. It is not as if without the Administrator's approval of each of the
statements, the statements cannot be posted by any of the members of the Group on the said platform.
An admin, by merely creating a group, does not publish or transmit material over the internet. Thus, the
Court drew a distinction between a mere group admin and an intermediary under the Act. The term
"intermediary" refers to any person who, on behalf of another person, receives, stores or transmits an
electronic record or provides any service with respect to that record. Generally, the admin of a group
does not store or receive electronic records on behalf of another. Rather, it is WhatsApp itself which is
arguably an intermediary as it acts as a medium between two or more persons.
(Extracted with requisite revisions and edits from ‘Can the administrator of a WhatsApp group be held
liable for objectionable posts?’ https://www.barandbench.com/columns/administrator-whatsapp-group-
liable-objectionable-posts)

74. Suresh created a whatsapp group for the preparation and discussion of CLAT related topics. Various
students were added in that group. One day, during one such discussion, the discussion got intense and
one Raju passed some sexually colored remark along with some obscene stickers against another group
member, Megha. Mr. Suresh observed all this but did not object to the remarks made by Raju and later
even after several requests by Megha, failed to remove him from the group. Can he now be held liable for
the act of Raju?
(a) Yes, he will be liable because he failed to remove Raju from the group despite several requests of
Megha after he passed colored remarks against her.
(b) Yes, since he did not object to such colored remarks by Raju, it indicates the presence of common
intention.
(c) No, he was neither involved in making such colored remarks nor supporting them when passed by
Raju.
(d) No, he cannot be held liable for acts of Raju because there was no common intention.

75. Two day ago, Raju and Suresh were studying and during the discussion they discussed the arrogance of
Megha and her habit/ want of always being correct and certain colored remarks were also made. Two
days later, the incident of Raju passing such remarks happened. Is Suresh liable for the content posted?
(a) Yes, his discussion two days ago indicates the presence of common intention.
(b) Yes, his discussion two days ago indicates the likelihood of existence of a pre-arranged plan.
(c) No, there was no common intention shared by Raju and Suresh for the comments passed or content
posted in the group.
(d) No, he is merely the admin of the group and created the group in good faith.

76. One of the stickers being shared by Raju in the group was previously shared in a personal chat with Raju
by the administrator of the group Suresh and hence it was argued that Suresh is also acting as an
intermediary in this situation and would be liable. Is the argument correct?
(a) Yes, Suresh has acted as the intermediary as the stored the sticker and shared it with Raju.
(b) No, Whatsapp is acting as the intermediary and not Suresh.
(c) Yes, Suresh has acted as the intermediary between Raju and Megha.
(d) No, Raju is the intermediary as he transmitted the sticker from Suresh to Megha.

Head Office: 127, Zone II, MP Nagar, Bhopal |+91-7676564400| https://www.toprankers.com Page 20 of 40
77. Bombay Police Act was recently amended to provide under Section 68 that “all persons shall be bound to
conform to the reasonable directions of a Police officer given in fulfillment of any of his duties under this
Act.” Directions against the whatsapp admin on posting of objectionable content by members of the
group were issued by the police officer. Would such directions be deemed reasonable?
(a) No, neither the group administrator’s approval is required nor he has any control over the statements
posted in the group by the participants.
(b) Yes, since the Bombay Police Act is enacted by the parliament it shall be deemed to have been well
contemplated and deliberated and thus be valid.
(c) No, the directions are unreasonable and actions are required to be undertaken against such
excruciating and ill-thought out directions.
(d) Yes, the Bombay Police Act has empowered the police to issue any directions as deemed
appropriate by them.

Passage (Q.78-Q.81): It is clarified that the fact of the petitioner having converted to Islam would not be
a relevant factor while ensuring that there is no interference in the liberty of the petitioners unless there is
any allegation by petitioner regarding forcible conversion. We, therefore, direct that the
administration/police authorities throughout the country will see to it that if any boy or girl who is a major
undergoes inter-caste or inter-religious marriage with a woman or man who is a major, the couple are not
harassed by any one nor subjected to threats or acts of violence, and anyone who gives such threats or
harasses or commits acts of violence either himself or at his instigation, is taken to task by instituting
criminal proceedings by the police against such persons and further stern action is taken against such
persons as provided by law. It is also clarified that petitioners, if major, are entitled to live together even if
not married and, therefore, proof of their marriage would not be required and shall not be insisted upon
by the concerned Police Officer.
(Extracted with requisite revisions and edits from ‘Can the administrator of a WhatsApp group be held
liable for objectionable posts?’ at https://www.barandbench.com/news/litigation/conversion-islam-
irrelevant-grant-protection-couple-allahabad-high-court)

78. Mr. Parvez (initially named Mr. Parma Chauhan) married Ms. Zoya Qureshi after converting into Islam.
They had initially eloped as their family was against these inter-religious ties. Parma later converted into
Islam and their nikah (marriage) was solemnized with all the Islamic rituals. They have been living
together for about 6 months and then their family got hold of their location. They started threatening them
of their lives as their relationship was affecting their family name. They approached the authorities for the
protection of their life and liberty. Are they entitled to any protection?
(a) Yes, it was a marriage between two adults and mere conversion is not relevant.
(b) No, there might be presence of forcible conversion which has not been ruled out and could only be
conclusively determined after some investigation is done.
(c) Yes, criminal proceedings should be instituted against the perpetrators.
(d) No, there were merely threats in the absence of any actual violence.

79. It was later brought into light by the mother of Parma Chauhan that there was forcible conversion
involved and hence the police authorities are not obligated to grant them any protection. The Police
agreed to the argument of her mother that forcible conversion was involved denied the protection sought
by the couple. Is the act of police justified?
(a) Yes, there is an allegation of forcible conversion of Mr. Parma.
(b) No, allegation of forcible conversion has not been made by Mr. Parma.
(c) Yes, the marriage is between adults who should be granted protection and conversion whether
forcible or not is immaterial.
(d) No, the police are under an obligation to grant protection to all adults irrespective of the existence of a
valid marriage.

Head Office: 127, Zone II, MP Nagar, Bhopal |+91-7676564400| https://www.toprankers.com Page 21 of 40
80. It was revealed that the marriage was contracted without the requisite number of witnesses, which is an
requisite under the Islamic laws and thus making the marriage irregular (also called fasid). Hence, when
they sought aid from the police to protect their interests and liberty the police denied any protection as
their marriage was not recognized even by their religion. Is the act of police justified?
(a) Yes, since the marriage is irregular according to Islamic laws.
(b) No, the protection is to be granted to every major and they are allowed to live together regardless of
marriage.
(c) Yes, the police are under an obligation to protect only those who have been under a proper wed-lock
and their liberty is at stake.
(d) No, the protection has to be granted to every individual as it has been mandated through various
judicial pronouncements.

81. They were granted protection at that time and 7 years passed after their marriage and they started living
in another locality. There was however certain threats by their new locality once they got to know that
there was an inter-religious marriage between the couple. Zoya approached the police to protect them
since such threats and take criminal action against those involved. Parma, this time, however admits that
his mother was right in her argument about the forced conversion and he did not speak about it due to
the political associations of Zoya. Would this new factor being disclosed by Parma, a relevant factor?
(a) No, there has already been a lapse of 7 years to raise any such contention.
(b) No, nobody can sleep over his rights.
(c) Yes, allegation of forcible conversion has been made by Parma and not anybody else.
(d) Yes, if the contentions of Mr. Parma turned out to be correct this will have a bearing on his life and
liberty.

Passage (Q.82-Q.85): Draft Rules released by the Supreme Court e-Committee on Monday for live-
streaming and recording court proceedings propose a 10-minute delay in transmission and exclusion of
communally sensitive cases and matters that involve sexual offences and gender violence against
women. Justice Chandrachud said the right of access to justice, guaranteed under Article 21 of the
Constitution, “encompasses the right to access live court proceedings”. The Rules intend to balance
between access to information and concerns of privacy and confidentiality.
Matrimonial matters, cases under the Protection of Children from Sexual Offences Act (POCSO) and
under the Juvenile Justice (Care and Protection of Children) Act would also be exempted from
livestream. The Bench can exempt, for reasons recorded in writing, any case it considers antithetical to
the administration of justice.
Personal information such as date of birth of parties, home address, identity card number, bank account
information, and the personal information of related parties, such as close relatives, witnesses and other
participants, will be deleted or muted during live-streaming.
“No person/entity (including print and electronic media, and social media platforms) other than an
authorised person/entity shall record, share and/or disseminate live-streamed proceedings or archival
data,” the Rules proposed.
(Extracted with requisite revisions and edits from ‘Draft Rules for live-streaming, recording of court
proceedings out’ at https://www.thehindu.com/news/national/draft-rules-for-live-streaming-recording-of-
court-proceedings-out/article34755473.ece)

82. Firm R&R was undergoing a dispute pertaining the assets of the company post its demerger under the
leadership of two of its founding partners. In the upcoming proceedings, there was a requirement for the
presentation of various documents providing the details of CSR activities of the company, its balance
sheets, and details of various tax related information all of which has been mandated to be provided in
the website of the company under the Companies Act, 2013. Post Covid and in pursuance of the draft
Rules, the Court decides to live stream the proceedings. One of the partners opposed this but his request
was denied by the Bench. Was the decision of the Bench flawed?
(a) Yes, business confidentiality will apply to the said hearing.

Head Office: 127, Zone II, MP Nagar, Bhopal |+91-7676564400| https://www.toprankers.com Page 22 of 40
(b) No, the discretion to exempt the live-streaming resides solely with the bench to cater to the
administration of justice.
(c) No, right to access of justice mandates the live streaming of the proceedings.
(d) No, the documents to be presented in the proceedings are already in the public domain (website) and
no business confidentiality will be applicable.

83. In another case before the trial court, a renowned celebrity Mr. Golden khan was accused to molestation
of a minor and booked under POCSO Act. The court decides the same to be held in-camera and no live
streaming of the same. Mr. Khan, however, approached the court and quoted Justice Chandrachud that
“right of access to justice, guaranteed under Article 21 of the Constitution, encompasses the right to
access live court proceedings.” Will his argument succeed?
(a) Yes, Article 21 being a Fundamental Right cannot be subject to any restrictions.
(b) No, the case involves the offence against a minor and POCSO Act’s applicability will prohibit the live
streaming of the same.
(c) No, it’s merely a statement by Justice Chandrachud and will not have any binding effect on any court
or its proceedings.
(d) Yes, administration of justice shall permit the live-streaming of such proceedings.

84. Despite several attempts, the proceedings were recorded and leaked in an online platform. It was
however put down from the platform. Few days later, a twitter user and law student, Ms. Sanjeevni posts
a clip from the said leak. A suit is brought against her. Is she liable for breach under the draft Rules?
(a) Yes, she is not an authorized person to record or share live-streamed proceedings.
(b) Yes, recording, sharing or disseminating of live-streamed proceedings is prohibited for everyone.
(c) Yes, the clip shared involves sharing of sensitive matter.
(d) No, the recording and initial sharing was done by a third party and she has only re-shared a part in
the clip.

85. One of the arguments given by Ms. Sanjeevni was on the premise that the purpose of her re-sharing of
the clip was educational involving a research on a particular field requiring the survey into the opinions of
twitter users and is not motivated by any commercial benefits. Will her argument hold well?
(a) Yes, the re-sharing of the clip was not in any print/electronic media platforms.
(b) Yes, the re-sharing was merely educational and not commercial.
(c) No, she was an unauthorized person to share or disseminate the clip.
(d) No, it was illegal to share a sensitive matter involving POCSO Act.

Passage (Q.86-Q.89): “Ignorance is no justification for normalizing any form of discrimination” said
Justice Anand Venkatesh in a judgment which issued a slew of guidelines to ensure the protection of
LGBTQIA+ persons in consensual relationships from police harassment. The Court observed that
legislative changes are needed to eradicate the social discrimination against LGBTQIA+ community and
to ensure the protection of their life and dignity. In this regard, the court mentioned how legislative
interventions are changing the attitude towards persons with disabilities and mental illness.
The hostilities they face are due to the fact that their relationship does not enjoy societal sanction. "...the
actual problem is not the fact that the law does not recognise a relationship but that the sanction that is
accorded by the society is not available. It is only for this reason, I strongly feel that the change must take
place at a societal level and when it is complemented by a law there will be a remarkable change in the
outlook of the society by recognising same-sex relationships.
Till the legislature comes up with an enactment, the LGBTQIA+ community cannot be left in a vulnerable
atmosphere where there is no guarantee for their protection and safety. This gap is now sought to be
filled in by way of issuing guidelines till law takes over and guarantees safety and protection.
(Extracted with requisite revisions and edits from 'My Upbringing Treated "Homosexual", "Gay", "Lesbian"
As Anathema' : Madras HC Judge Explains How He Overcame Prejudice Against LGBTQIA+ Community
at https://www.livelaw.in/top-stories/lgbtqia-homosexual-gay-lesbian-madras-hc-anand-venkatesh-
overcome-prejudice-175342)

Head Office: 127, Zone II, MP Nagar, Bhopal |+91-7676564400| https://www.toprankers.com Page 23 of 40
86. Ms. Eena and Ms. Meena, a lesbian couple decided to elope from their parents who were unsupportive of
their relationship. They through the help of an NGO managed to obtain an accommodation and were
living there for about 4 months. However, their parents filed missing complaints of those girls. The police
tried to bring them to their families but they resisted the authorities. They approached the Court for relief
from what they called police harassment. Should the Court grant them any relief?
(a) No, there exists no case of discrimination against the couple.
(b) No, the case in question is about the missing complaints of the girls and is independent of any police
harassment or discrimination.
(c) Yes, ignorance of law by the police and family is no excuse for harassment.
(d) Yes, the Court should ensure the protection of their life and dignity.

87. The Landlord of their apartment filed a police complaint for their eviction as after giving them notice for
payment of rent of last 2 months they failed to comply. The couple however alleged that this eviction
amounts to discrimination on the ground of their sexual orientation as the landlord was not favorable of
same sex relationships. In a suit brought against such eviction, will their argument hold well before the
Court?
(a) Yes, they have been discriminated on the ground of their sexual orientation.
(b) No, there is no discrimination on the basis of their sexual orientation and lack of societal sanction.
(c) Yes, their relationship lacked societal sanction from the landlord and the police.
(d) No, there is no police harassment against them.

88. Before they found this accommodation, they were denied accommodation by multiple people. One such
person was Mr. Gangadhar who denied them the accommodation as according to him even law doesn’t
recognize same-sex relationship and hence even his neighbors denied renting them accommodation. Is
any liable for this conduct?
(a) Yes, Ignorance of law is no excuse.
(b) Yes, Ignorance cannot rationalize any form of discrimination.
(c) No, Societal sanction has already been ensured through legal frameworks and individual
discrimination is not a ground of relief.
(d) Yes, Section 377 IPC has been read down by the Apex Court.

89. During their visit to a temple, they were denied to perform certain rituals as a couple as they believed that
it would anger the sacred deities and bring havoc on Earth and were thus forcibly removed. A suit was
brought against such discrimination. They pleaded their independence to manage their affairs as a
Freedom guaranteed under Fundamental Rights which cannot be interfered with in the absence of any
legislative frameworks existing. Will their argument succeed?
(a) Yes, Freedom to manage religious affairs is guaranteed under Article 26 of the Constitution.
(b) Yes, no legislative framework exists to ensure them the liberty to exercise such a freedom.
(c) No, despite the absence of any legislation to this effect they cannot be left abandoned without any
protection and safety.
(d) No, society should evolve and give sanction to their relationship.

Head Office: 127, Zone II, MP Nagar, Bhopal |+91-7676564400| https://www.toprankers.com Page 24 of 40
Passage (Q.90-Q.93): Section 16 of the U.P. Sugarcane (Regulation of Supply and Purchase) Act, 1953
is not repugnant to Section 3(2)(c) of the Essential Commodities Act, 1955 and Clause 3 of the
Sugarcane (Control) Order, 1966 as, as observed hereinabove, the price which is fixed by the Central
Government is the “minimum price” and the price which is fixed by the State Government is the “advised
price” (or remunerative price) which is always higher than the “minimum price” fixed by the Central
Government and therefore, there is no conflict. It is only in a case where the “advised price” fixed by the
State Government is lower than the “minimum price” fixed by the Central Government, the provisions of
the Central enactments will prevail and the “minimum price” fixed by the Central Government would
prevail. So long as the “advised price” fixed by the State Government is higher than the “minimum price”
fixed by the Central Government, the same cannot be said to be void under Article 254 of the
Constitution of India.
(Extracted with requisite revisions and edits from ‘State Government cannot fix the “minimum price” of
sugarcane once Centre has already fixed it’ at https://www.scconline.com/blog/post/2020/04/24/state-
government-cannot-fix-the-minimum-price-of-sugarcane-once-centre-has-already-fixed-it/)

90. Central Government having exercised the power under Entries 33 and 34 List III of seventh Schedule
fixed the “minimum price” of jute at Rs. 6000/ 100kgs. However, the State Government owing to the
pandemic decided to decrease the minimum price to Rs. 5500/ 100kgs. The same was brought into
question before the court of law. Will the action against the State Government succeed?
(a) Yes, advised price cannot be lower than the minimum price fixed by the Central Government.
(b) No, by virtue of List III, State Government is empowered to fix the prices.
(c) No, State Government has fixed the price on the basis of pandemic.
(d) Facts are insufficient.

91. State Government of UP set the price of sugarcane @ Rs 35/ Kg which was lesser than the price set by
the Central Government. They argued that under since it is a subject under Concurrent list; it is at an
equal pedestal in setting the minimum price for the sugarcanes and their pricing shall prevail. Will their
argument succeed?
(a) No, advised price cannot be lower than the minimum price fixed by the Central Government.
(b) Yes, List III empowers the State Government to fix the prices.
(c) No, Central Government has the monopoly in fixing of prices of commodities like sugarcane.
(d) No, Prices fixed by Central Government cannot be altered by the State Government.

92. State Government of UP argued that there exist a repugnancy under List III (as well as under the State
Laws) as both Centre and State can set the minimum price at which sugarcane can be brought or sold. Is
there argument True or False?
(a) True.
(b) False.
(c) Partially true.
(d) Cannot be determined.

93. The remunerative price of sugarcane was set @ Rs 40/ Kg for the season. However, later the Central
Government decided to increase the minimum price to Rs 50/ kg due to the failure of crops in many parts
of the country. The State Government of UP also changed the price to Rs. 45/ Kg. It was disputed that
since the price notified by the State Government is through a later notification, such a pricing should
prevail over the minimum price set by the Central Government. Is this argument correct?
(a) Yes, the State Government’s advised price is through a later notification and hence it should prevail.
(b) No, Central Government has the authority of setting the minimum price.
(c) No, the price set by State Government is lower than the minimum price; the price fixed by the Central
Government shall prevail.
(d) Yes, State Government can set the remunerative price as per its discretion.

Head Office: 127, Zone II, MP Nagar, Bhopal |+91-7676564400| https://www.toprankers.com Page 25 of 40
Passage (Q.94-Q.97): No fundamental right is conferred on any religion to degrade the other religion. I
have already pointed out that while professing any religion, the religious heads or professing by any
person should not degrade other religion. No doubt, while setting the law in motion invoked Section
295(A) of IPC regarding deliberate and malicious acts, intended to outrage religious feelings of any class
by insulting its religion or religious beliefs and after the investigation, the Investigating Officer invoked
Section 298 of IPC (uttering words to hurt religious feelings) uttering, words, etc. with deliberate intention
to wound the religious feelings of any person. Having taken note of the complaint averments and also the
statement of witnesses, with deliberate intention to wound the religious feelings of other religion words
are uttered while propagating.
(Extracted with requisite revisions and edits from ‘No religion has fundamental right to degrade other
religions: Karnataka High Court rejects plea by 2 Christians accused of insulting Quran, Gita’ at
https://www.barandbench.com/news/litigation/fundamental-right-religion-karnataka-high-court-christian-
quran-bhagavad-gita)

94. Ms. Mary during a religious procession was heard using the statement that “Jesus created the world and
can destroy it when he wishes and even the greatest of saint cannot stop the ragnarok”. A case for
hurting the religious sentiments was brought against her. Is she liable?
(a) No, there is no degradation of other religions in the statement of Ms. Mary.
(b) Yes, there is degradation of all other religions by virtue of such a statement.
(c) No, she only professed the idea of the creation and destruction of the world without any reference to
any specific religion.
(d) Yes, a malicious statement is made to outrage the sentiments of other religions.

95. Ms. Mary in the past had also elicited a lot of controversy when she said that "only Bible can tell the
future and no other religious scripts give any information and tsunami is coming in the future." Should she
be made liable for such a statement?
(a) No, there is no degrading of other religions as no specific religion has been targeted and a general
statement has been made.
(b) Yes, religious feeling will be affected irrespective of the intention involved while making such a
statement.
(c) No, merely telling the coming of tsunami or telling of future doesn’t mean degradation of other
religion.
(d) Yes, professing one religion should not be at the cost of degrading other religions.

96. Another leader of a religious group during a gathering said “Sabka Malik Ek (There is only one God) who
is the supreme deity and its time when everyone learns to identify the real god abandoning all the beliefs
from those phony scriptures”. Should any liability be attributed to him?
(a) No, there is no degrading of other religions as no specific religion has been targeted and a general
statement has been made.
(b) Yes, professing one religion should not be at the cost of degrading other religions.
(c) Yes, one cannot identify as to who can be called the Supreme Deity and it will invoke religious
turmoil.
(d) No, no particular religion has been professed through such a statement.

97. A statement “Neither Bhagavad Gita nor Quran will provide any peace of mind or come to the rescue of
any person and it is only the Bible which provides the most authentic and pragmatic view of the creation
of the world”. Whether this will be struck by Section 295(A) or Section 298 of IPC?
(a) Yes, such a statement is degrading and will result in insulting of other religious beliefs.
(b) No, such a statement is incapable of wounding the religious feelings of others.
(c) Yes, the statement is targeted as a result of hatred towards other religions.
(d) No, it is a general statement without any substance devoid of instilling any effect/ opposition by
followers of other religions.

Head Office: 127, Zone II, MP Nagar, Bhopal |+91-7676564400| https://www.toprankers.com Page 26 of 40
Passage (Q.98-Q.102): The Delhi HC ruled that photographs taken from social media and uploaded on
pornographic websites without the consent of the concerned person amounts to an offence under Section
67 of the IT Act. The act, regardless of the photograph being offensive/obscene, amounts to a breach of
a person's privacy; the court may in appropriate cases pass an order of restrain.
The court also observed the role of intermediaries, and held that they are mandated to disable access to
such content once they receive a notification by the appropriate government or its agency. If the
intermediary fails to do so, they lose the exemption from liability.
The court said, "In the first instance, an intermediary cannot say that it is unable to remove/disable
access to offending content despite knowledge as contemplated in law,”
An intermediary is now required to inform its users that they must not host, display, upload modify,
publish, transmit, store, update or share any information that 'belongs to another person and to which the
user does not have any right' being 'invasive of another's privacy’,”
The court observed that a time-bound grievance redressal mechanism is provided of i.e. 24 hours from
the receipt of a complaint given to intermediary to 'take all reasonable measures to remove or disable
access to such content.
The law enforcement body should be directed to obtain all unique user information relating to the
offending content, and the same must be not later than 72 hours.
Source: https://www.thequint.com/news/india/delhi-hc-issues-directions-on-removal-of-offensive-web-
content

98. Rachit is a sophomore in College. He had recently attended a party in college where he had clicked
several pictures with his friends and batch-mates. He later uploaded several of those pictures on his
social media handles, out of which, one was of him and a classmate Rahul. Rahul decided to prank
Rachit and took the picture of both of them and uploaded on a pornographic website. This infuriated
Rachit, and now he wants to take legal actions against Rahul. Will his action sustain?
(a) Rachit cannot take any action against him because that picture had both of them in it.
(b) Rachit can take legal actions against him because it was uploaded without his consent
(c) Rachit cannot take any legal actions because the picture wasn’t per se obscene or offensive, and was
merely a picture taken at a party.
(d) Rachit can take legal actions because the picture was taken from his social media.

99. Apoorva is a web designer in WebFor Solutions. She recently got into a heated argument with one of her
colleagues, Manoj, on bringing some new changes to their website. In the heat of the argument, she
called Manoj an “uptight idiot” in front of several other staff members. Embarrassed by this, Manoj went
home, took one of the pictures from Apoorva’s LinkedIn profile, morphed the face of a frog on it and
shared it on a pornographic website. Apoorva came to know about this and when she confronted Manoj
on this and threatened to take legal actions against him, Manoj defended himself saying that her face is
not visible in the picture, thus, he hasn’t committed any offence.
(a) Manoj’s argument is valid because no one can identify it’s Apoorva in the picture as her face is not
visible.
(b) Manoj’s argument is invalid because he morphed the picture of a frog onto her picture without her
consent.
(c) Manoj’s argument is valid because if no one can identify Apoorva in that picture, that would not
amount to breach of privacy.
(d) Manoj’s argument is invalid, because he took Apoorva’s picture, morphed it and uploaded it on the
website without her consent.

Head Office: 127, Zone II, MP Nagar, Bhopal |+91-7676564400| https://www.toprankers.com Page 27 of 40
100. Considering in the above question, had Manoj not morphed her picture but had he just written her name
on the picture of a frog eating flies, would that have warranted the attention of the Act?
(a) Yes, as he had defamed her which amounted to offensiveness under the act.
(b) No, as Manoj did not use her picture or private information but had just used a common name.
(c) No, as the picture did not contain any offense or obscene pictography.
(d) Yes, as he did so with the intention of implicating her.

101. Akash, an engineering student, uses the Social Media platform to voice his opinions on various
controversial topics. Once, when the discussions around the deportation of the Rohingya Muslims were
going on, he used twitter to protest against this virtually, calling out the Government for being ‘insensitive’
and ‘hypocrite’. Soon, he became the subject-matter of several memes. In one of the memes his face
was morphed into a picture with a man standing cloth-less, covering himself with a stack of cash while
the Indian Flag was being burned. The meme was posted as stories on Photogram and would disappear
after 24 hours. Akash complained the Cyber Cell regarding this who then instructed Instagram to take
down the content. Photogram did not take any action to take down those stories thinking that they would
eventually disappear after 24 hours. By this, would Photogram be exempted from any future liability?
(a) Yes, because all the memes were posted as stories and thus, would eventually disappear after 24
hours.
(b) No, because it did not fulfil the mandate of taking down the photo immediately after being asked to do
so.
(c) Yes, because the purpose of the new provisions of law is to take down all the offensive content and in
this case, the photos would automatically disappear, thereby fulfilling the purpose.
(d) No, because it allowed an offensive meme to be uploaded.

102. Considering in the above question, had Photogram not taken cognizance of the memes due to their third
party encryption policy which allows users to have complete control over what they post or see, without
letting the application monitor such content,. However, they do have an override feature which can be
used, but would open the application up to hackers and subsequent lawsuits. Would they still be liable for
failure to remove the content?
(a) They will be liable to remove such content from their portal as soon as they receive such notification.
(b) They will be liable to remove the content regardless of the consequences.
(c) They cannot subject their users to malware/ransom ware attacks, and thus, would not be liable.
(d) Photogram uses third-party encryption and thus should be exempted from the laws as they do not
mention these laws.

Head Office: 127, Zone II, MP Nagar, Bhopal |+91-7676564400| https://www.toprankers.com Page 28 of 40
SECTION - D: LOGICAL REASONING

Passage (Q.103-Q.107): The Indian agricultural economy is made up of dazzlingly complex market
linkages connecting approximately 263.1 million agricultural workers, directly or indirectly, to over 1 billion
domestic consumers – and many other consumers around the world. When the Central Government
announced a 21-day national lockdown in response to COVID-19 on March 24th, there were deep fears
that the agricultural sector would be crippled. Many worried that the transportation restrictions in
particular would bring to a crashing halt the highly intertwined and labour-intensive food systems that
deliver agricultural goods across the country.

Yet despite the challenges of COVID-19, these formal and informal networks seemed remarkably robust
on the surface – at least at first. Following the initial lockdown, while the economy grounded to a halt,
food prices stabilized for many agricultural goods to pre-lockdown levels – though food delivery to state-
regulated agricultural markets (mandis) plummeted. By June, just three months after the lockdown was
first imposed, the arrival of goods to mandis had rebounded to rates comparable to prior years.

However, this rebound in market arrivals did not persist past Phase 5 of India’s Central Government
response to the pandemic in June (otherwise known as Unlock 1.0) – and a deeper investigation into
agricultural sector data highlights latent, systematic challenges. In addition to a decrease in food arrivals
to mandis, reports of farmer suicides and data on major food security challenges suggest that agricultural
markets may not be as healthy as market figures suggest.
Source:https://intelligence.weforum.org/monitor/latest-knowledge/c97681a901c0401c8d9425285b506b24

103. The author is arguing for…?


(a) Economic condition of pre and post COVID-19.
(b) Impact on Indian agricultural due to COVID-19.
(c) Lockdown condition in India due to COVID-19.
(d) Farmers are highly affected due to COVID-19.

104. Which of the following option cannot be inferred from the passage?
(a) Agricultural sector is one of the most important sectors in India.
(b) Agricultural sector is expected to be affected exceptionally.
(c) Agricultural sector is the only sector which is affected largely.
(d) All the above.

105. Which one of the following, if true, may not help to explain that this rebound in market arrivals did not
persist past Phase 5?
(a) Because of systematic challenges.
(b) Because of decrease in food arrival to mandis.
(c) Agricultural market is not as healthy as market figure.
(d) None of the above.

106. Which one of the following, if true, most undermines the argument?
(a) Transportation sector depends on agricultural sector.
(b) Due to restrictions arrival of food at mandis delayed.
(c) Food commodities arrival has fallen post phase 5.
(d) State-regulated agricultural markets saw more influx post unlock 0.1

Head Office: 127, Zone II, MP Nagar, Bhopal |+91-7676564400| https://www.toprankers.com Page 29 of 40
107. Which of the following statement act as an evidence to show agricultural commodities delivery has
reduced in mandis during COVID-19?
(a) 263.1 million agricultural workers, directly or indirectly depend over 1 billion domestic consumers –
and many other consumers around the world.
(b) Rebound in prices did not persist past Phase 5 of India’s Central Government response to the
pandemic
(c) 26% of households reported having to limit food portion sizes or reduce meals due to non-availability.
(d) 90% of all our survey respondents with ration cards received rice, wheat or pulses (legumes) for free
from Public Distribution Shops in June.

Passage (Q.108-Q.112): Painting, the expression of ideas and emotions, with the creation of certain
aesthetic qualities, in a two-dimensional visual language. The elements of this language—its shapes,
lines, colours, tones, and textures—are used in various ways to produce sensations of volume, space,
movement, and light on a flat surface. These elements are combined into expressive patterns in order to
represent real or supernatural phenomena, to interpret a narrative theme, or to create wholly abstract
visual relationships. An artist’s decision to use a particular medium, such as tempera, fresco, oil, acrylic,
watercolour or other water-based paints, ink, gouache, encaustic, or casein, as well as the choice of a
particular form, such as mural, easel, panel, miniature, manuscript illumination, scroll, screen or fan,
panorama, or any of a variety of modern forms, is based on the sensuous qualities and the expressive
possibilities and limitations of those options. The choices of the medium and the form, as well as the
artist’s own technique, combine to realize a unique visual image.

Earlier cultural traditions—of tribes, religions, guilds, royal courts, and states—largely controlled the craft,
form, imagery, and subject matter of painting and determined its function, whether ritualistic, devotional,
decorative, entertaining, or educational. Painters were employed more as skilled artisans than as creative
artists. Later the notion of the “fine artist” developed in Asia and Renaissance Europe. Prominent painters
were afforded the social status of scholars and courtiers; they signed their work, decided its design and
often its subject and imagery, and established a more personal—if not always amicable—relationship
with their patrons.

Source: https://www.britannica.com/art/painting

108. Which one of the following most accurately expresses the conclusion drawn by the passage as a whole?
(a) Earlier cultural traditional people employed painters as a skilled artisan rather than creative artisans.
(b) Painting, the expression of ideas and emotions, with the creation of certain aesthetic qualities, in a
two-dimensional visual language.
(c) Elements are combined into expressive patterns in order to represent real or supernatural
phenomena.
(d) An artist’s decision to use a particular medium is based on the sensuous qualities and the expressive
possibilities.

109. According to the passage, the statement “Painters were employed more as skilled artisans than as
creative artists.” implies that:
(a) In early ages painters were more skilled rather than creative.
(b) In early ages painters were both skilled and creative.
(c) In early ages more skilled painters were required rather than creative.
(d) In early ages more skilled as well as creative painters were required rather than just skilled or
creative.

Head Office: 127, Zone II, MP Nagar, Bhopal |+91-7676564400| https://www.toprankers.com Page 30 of 40
110. Which one of the following, if true, most strengthens the author’s reasoning?
(a) Painting weakened under Renaissance humanism.
(b) The range and interpretation of subjects in different forms of devotional painting express a particular
attitude to the relationship between human beings and God.
(c) Poetry and literature are found in Eastern miniatures and Asian scrolls.
(d) Painting speaks the language and emotions of people in the form of visual art.

111. Which one of the following is an assumption required by the argument “Painters were employed more as
skilled artisans than as creative artists”?
(a) In early days there was no value of creative painters.
(b) In early days there was no value of skilled painters.
(c) Skilled painters are more capable to paint ritualistic, devotional, decorative, entertaining, or
educational painting than creative painting.
(d) There is no need of skilled painters to paint ritualistic, devotional, decorative, entertaining, or
educational painting.

112. Which of the following reasoning can be true to show that modern paintings required more creative
painters rather than skilled painters?
(a) Modern painting ideas are not limited to ritualistic, devotional, decorative, entertaining, or educational
purposes only.
(b) Modern paintings are used for business purposes.
(c) Modern painting has no meaning of rituals, devotion, entertainment, or education in modern lives.
(d) All of the above.

Passage (Q.113-Q.117): Globalization refers to integration between people, companies, and


governments. Most noteworthy, this integration occurs on a global scale. Furthermore, it is the process of
expanding the business all over the world. In Globalization, many businesses expand globally and
assume an international image. Consequently, there is a requirement for huge investment to develop
international companies.
First of all, Foreign Direct Investment (FDI) increases at a great rate. This certainly is a huge contribution
of Globalization. Due to FDI, there is industrial development. Furthermore, there is the growth of global
companies. Also, many third world countries benefit from FDI.
Technological Innovation is another notable contribution of Globalization. Most noteworthy, there is a
huge emphasis on technology development in Globalization. Furthermore, there is also technology
transfer due to Globalization. The technology would certainly benefit the common people. The quality of
products improves due to Globalization. This is because manufacturers try to make products of high-
quality. This is due to the pressure of intense competition. If the product is inferior, people can easily
switch to another high-quality product.
To sum it up, Globalization is a very visible phenomenon currently. Most noteworthy, it is continuously
increasing. Above all, it is a great blessing to trade. This is because it brings a lot of economic and social
benefits to it.
Source: https://www.toppr.com/guides/essays/globalization-essay/

113. Which one of the following is most strongly supported by the information above?
(a) Globalisation increases the competition in every business sector.
(b) Innovation in technology increases due to globalisation.
(c) People choose inferior products if they are cheap.
(d) All of the above.

Head Office: 127, Zone II, MP Nagar, Bhopal |+91-7676564400| https://www.toprankers.com Page 31 of 40
114. Which one of the following most accurately expresses the overall conclusion of the passage?
(a) Globalization is a very visible phenomenon in today’s world.
(b) Technology promotes globalisation.
(c) FDI has a huge contribution of globalisation.
(d) Globalisation brings a lot of economic and social benefits to it.

115. Which one of the following, if assumed, enables the conclusion of the argument to be properly inferred?
(a) High quality product can only be made by intense competition.
(b) If there is no globalisation then there is no competition in a country.
(c) Globalisation incites competition and that in turn increases quality at minimum price.
(d) Industry may begin to thrive in LEDCs at the expense of jobs in manufacturing in the UK

116. Which one of the following, if true, most undermines the conclusion of the passage?
(a) Globalisation is viewed by many as a threat to the world's cultural diversity.
(b) Globalisation helps in sharing of ideas, experiences and lifestyles of people and cultures.
(c) Globalisation may help to make people more aware of global issues such as deforestation and global
warming and alert them to the need for sustainable development.
(d) Globalisation increases awareness of events in faraway parts of the world.

117. Which of the following can be the correct premises or evidence of the statement “Globalisation helps in
reaching out to better technology”?
(a) India imports 70% of automobile parts from Germany.
(b) India does not allow FDI in every sector.
(c) India and USA partnered up in many defence missions.
(d) Domestic market in India is very inefficient in production.

Passage (Q.118-Q.124): Beyoncé: Lady Macbeth is just another victim in William Shakespeare’s
famous tragedy and is entirely blameless for the disastrous events that unfolded. This is because, like
Macbeth, she is under the curse of the three witches (the real villains of the play) and is therefore
controlled by them. Throughout the story, Lady Macbeth talks about destiny and fulfilling a prophecy,
rather than making decisions for herself. Lady Macbeth is good, according to her supporters, as she does
not actually murder anyone, unlike Macbeth. The worst thing she does is provide a sleeping potion in a
jug of wine for the king’s guards, which is hardly a serious crime. Instead of being a killer, she is simply
being a dutiful wife and supporting her husband’s dearest wish to be King. Even when things are doomed
for Macbeth, she is loyal to him right until the end.

Shakira: Lady Macbeth is an evil character, who is completely in control of not only her own actions but
also her husband’s which makes it a tragedy. Lady Macbeth although does not murder the king, it is her
idea to kill him. To make matters worse, her idea is premeditated, because she has already poisoned the
king’s guards when she tries to persuade her husband to kill him. Would Macbeth have actually gone
through with his crimes, had she not been so forceful with her venomous powers of persuasion? Some
people believe that Lady Macbeth certainly is not a dutiful wife who wants the best for her husband, but
instead is full of self-ambition. Had she wanted the best for him, would she have put him at risk of being
accused of treason?
Adjudicator: As you can see, there are strong arguments for both viewpoints and therefore no one can
deny that Lady Macbeth is indeed a divisive character.

118. Which of the following statements can be concluded with surety from the conversation above?
(a) Both Macbeth and Lady Macbeth are to be blame for the death of the king
(b) That the king was killed
(c) Both Macbeth and Lady Macbeth are evil characters
(d) That the witches were real murderers of the king

Head Office: 127, Zone II, MP Nagar, Bhopal |+91-7676564400| https://www.toprankers.com Page 32 of 40
119. Which of the following statements if true would most substantially weaken Beyoncé’s argument?
(a) The prophecy was for Macbeth and not Lady Macbeth
(b) Lady Macbeth actually asked Macbeth to repent for his crimes and stop any further bloodshed
(c) Lady Macbeth actually went to kill the king herself before Macbeth went, but failed.
(d) Lady Macbeth like all the other noble women of the kingdom just wanted to be the queen.

120. Which of the following statements if true would most substantially weaken Shakira’s argument?
(a) The three witches just after making the prophecy sent an enchanted ring to Lady Macbeth which she
always wore and which completely changed her after that.
(b) Lady Macbeth always supported Macbeth even though she knew that those were criminal acts
(c) Both Macbeth and Lady Macbeth continued to fulfil their thirst for power even after becoming the king
and queen.
(d) No one would have died if Lady Macbeth wouldn’t have asked Macbeth to go to the witches again

121. Which among the following statements is NOT true based on the information given in the passage
above?
(a) At least one person considers Lady Macbeth to be evil
(b) Not all consider Lady Macbeth to be evil
(c) Everyone considers Lady Macbeth to be evil
(d) Some consider Lady Macbeth to be evil

122. To which of the following points will both the speakers in the above passage agree?
(a) Lady Macbeth had a change in attitude in the previous and later part of the play.
(b) Macbeth is a tragedy
(c) Lady Macbeth is innocent
(d) The king was an idiot

123. Which among the following statements, if true, would have no effect on either Beyoncé’s or Shakira’s
arguments?
(a) Fate is responsible for the things that happen
(b) Whatever happened was the revenge of the three witches for their insult by Macbeth
(c) The prophesy was that Macbeth would be king that would have happened anyways
(d) It’s nothing new that all nobles and their wives wanted to become kings and queens and would do
anything to achieve it.

124. The statement, “As you can see, there are strong arguments for both viewpoints and therefore no one
can deny that Lady Macbeth is indeed a divisive character”
(a) Forms assumption for the passage
(b) Forms conclusion for the passage
(c) Forms premise for the passage
(d) Weakens the overall argument of the passage.

125. 3, 4, 10, 33, 136, 685, ?.


(a) 3410 (b) 4802 (c) 5145 (d) 4116

126. In a certain code, '147' means 'spread red pen'; '156' means 'dust one pen' and '164' means 'one red
pen'. Which digit in that code means 'dust'?
(a) 2 (b) 3 (c) 5 (d) 6

127. Which set of letters when sequentially placed at the gaps in the given letter series shall complete it?
m_ _NO_n_pMN_P_
(a) mNmoQO (b) nMmoOQ (c) mnQOPo (d) mNmoOQ

Head Office: 127, Zone II, MP Nagar, Bhopal |+91-7676564400| https://www.toprankers.com Page 33 of 40
128. In an examination in English, Harsh obtained more marks than total of marks obtained by Kunal and
Debu. The total of marks obtained by Kunal and Shankar were more than Harsh. Sonal obtained more
marks than Shankar. Neha obtained more marks than Harsh. Who amongst them obtained highest
marks?
(a) Harsh (b) Neha (c) Sonal (d) Data inadequate

129. If GLUED is coded as 142442108, then how will START be coded as?
(a) 384023640 (b) 046320483 (c) 192011820 (d) 028110291

Directions (Q.130): In each question below, three statements are given followed by two conclusions
numbered I and II. You have to take the given statements to be true even if they seem to be at variance
with commonly known facts. Read all the conclusions and then decide which of the given conclusions
logically follow(s) from the given statements, disregarding the commonly known facts.

130. Statements:
Some alive are dead.
All dead are souls.
No soul is human.
Conclusions:
I. Some humans being alive is a possibility.
II. No dead is human.
(a) If both conclusions I and II follow. (b) If only conclusion I follows.
(c) If only conclusion II follows. (d) If either conclusion I or II follows.

Direction (Q.131): Some statements are given below followed by some conclusions. Taking the given
statements to be true even if they seem to be at variance from commonly known facts, read all the
conclusions and then decide which of the given conclusion logically follows the given statements.

131. Statements:
I. All guavas are fruits.
II. No carrot is a fruit.
Conclusions:
I. Some carrots are guavas.
II. All guavas are carrots.
III. No guava is a carrot.
(a) Only conclusion (I) follows (b) Only conclusion (II) follows
(c) Only conclusion (III) follows (d) No conclusion follows

132. 7 persons A, B, C, D, E, F and G has different age. A is older than B, who is older than only one person.
C is younger than D but older than G. E is younger than G but older than A. Who is the youngest among
all?
(a) A (b) B (c) F (d) G

Head Office: 127, Zone II, MP Nagar, Bhopal |+91-7676564400| https://www.toprankers.com Page 34 of 40
Directions (Q.133 and Q.134): At a party seven friends are sitting on seven chairs arranged in a row.
The friends are: Suresh, Neha, Mukesh, Ravi, Kashif, Mayank and Prateek and they are sitting not
necessarily in the same order. It is also known that:
(i) Neha is sitting beside Kashif.
(ii) Prateek is the best friend of the party’s host and so he is sitting in the middle of the row.
(iii) Either of Suresh or Ravi, but not both, is sitting at one of the ends of the row.
(iv) Mukesh and Mayank are sitting as far as possible from each other, keeping in mind all the conditions
of the problem.

133. Which of the following represents the number of people who could not be sitting between Mukesh and
Suresh?
(a) 0 (b) 2 (c) 3 (d) 5

134. Which two of the friends, among the given pairs, cannot sit adjacent to each other?
(a) Mayank and Ravi
(b) Mukesh and Neha
(c) Ravi and Kashif
(d) Each of the given pairs in the first three options could be sitting next to each other.

Head Office: 127, Zone II, MP Nagar, Bhopal |+91-7676564400| https://www.toprankers.com Page 35 of 40
SECTION - E :QUANTITATIVE TECHNIQUES

Directions (Q.135-Q.140): Read the following information carefully and answer the given questions.

Radhe travels (A) km distance at the speed of Z km/hr and reaches his office in 2 hours. If he increases
his speed by 20%, he reached (B) hours less than the previous. Argya travels twice the distance travelled
by Radhe and reached his office in 6 hours at the speed of40 km/hr . Radhe carries a bag which contains
(x) grey ball, 5blue ball and 10 purple balls. He takes one ball randomly and the probability of getting a
purple ball is1/2. Argya also carries a bag which contains (x+1) skycaps, (x-1) white caps and 5 brown
caps. He also takes one cap and the probability of getting a brown cap is (C).Radhe’s mother bought a
cycle with a discount of 20%on labeled price and he again marks up 25%. She offers at 15% discount
and sold it to Radhe’s friend and gained (D)%, if the labeled price of the cycle is Rs.40000.

135. Find the value of (A)


(a) 240 (b) 180 (c) 120 (d) 160

136. Find the value of (B)


(a) 24 mins (b) 20 mins (c) 12 mins (d) 36 mins

137. Find the value of (x)


(a) 3 (b) 4 (c) 2 (d) 5

138. Find the value of (C)


(a) ½ (b) 1/3 (c) 1/5 (d) 1/4

139. Find the value of (D)


(a) 8.75% (b) 13.5% (c) 15% (d) 6.25%

140. Find the value of (Z)


(a) 60 (b) 72 (c) 44 (d) 50

USE FOR ROUGH WORK

Head Office: 127, Zone II, MP Nagar, Bhopal |+91-7676564400| https://www.toprankers.com Page 36 of 40
Directions (Q.141-Q.145): The following information is about performance of Tamanna in an exam.
Read the information carefully and answer the following question.

The exam consists of 200 marks, with 5 sections i.e. Physics, Math, English, BIOLOGY., Chemistry.
3
Tamanna attempted 22 questions in Physics with an accuracy of 7711 %. Each question of physics
consists of 2 marks with a negative marking of 25%. (if right question is of 2 mark, then 0.5 mark will be
deducted for each wrong answer).
Each section of the exam have the 25% of negative marking for each wrong question. The total number
of questions in physics is 30. Each question of Chemistry consists of 1/2 marks and maximum marks in
Chemistry are 10. Total 16 questions are attempted by Tamanna in Chemistry with the ratio of right
questions to wrong questions 3 : 1.
The number of questions in English is equal to maximum marks of English. Tamanna attempted 26
questions with 50% accuracy. The number of questions attempted in English is 65% of the total number
of questions in English.
BIOLOGY section consists of 40 questions with each question 0.75 marks. Tamanna attempted 23
questions out of which 8 are wrong. Math section contains 40 questions out of which Tamanna attempted
35 questions and got 52.5 marks.

141. Another student radhe attempted 70% questions in the same exam, then find the number of questions
left by radhe.
(a) 119 (b) 68 (c) 51 (d) 65

142. Find the marks obtained by Tamanna in BIOLOGY.


(a) 8.75 (b) 9.25 (c) 9.75 (d) 10.75

143. The number of correct questions in physics is how much more than the number of incorrect questions in
the same subject?
(a) 12 (b) 7 (c) 18 (d) 9

144. Find the total marks obtained by Tamanna in the exam.


(a) 101 (b) 105 (c) 109 (d) 102

145. Find the total number of incorrect questions attempted by Tamanna in the exam.
(a) 27 (b) 15 (c) 28 (d) 37

USE FOR ROUGH WORK

Head Office: 127, Zone II, MP Nagar, Bhopal |+91-7676564400| https://www.toprankers.com Page 37 of 40
Directions (Q.12-Q.16): Study the following table and answer the questions based on it.
Expenditures of a Company (in Lakh Rupees) per Annum Over the given Years.
Item of Expenditure
Year
Salary Fuel and Transport Bonus Interest on Loans Taxes
1998 288 98 3.00 23.4 83
1999 342 112 2.52 32.5 108
2000 324 101 3.84 41.6 74
2001 336 133 3.68 36.4 88
2002 420 142 3.96 49.4 98

146. What is the average amount of interest per year which the company had to pay during this period?
(a) Rs. 32.43 lakhs
(b) Rs. 33.72 lakhs
(c) Rs. 34.18 lakhs
(d) Rs. 36.66 lakhs

147. The total amount of bonus paid by the company during the given period is approximately what percent of
the total amount of salary paid during this period?
(a) 0.1% (b) 0.5% (c) 1% (d) 1.25%

148. Total expenditure on all these items in 1998 was approximately what percent of the total expenditure in
2002?
(a) 62% (b) 66% (c) 69% (d) 71%

149. The total expenditure of the company over these items during the year 2000 is?
(a) Rs. 544.44 lakhs (b) Rs. 501.11 lakhs
(c) Rs. 446.46 lakhs (d) Rs. 478.87 lakhs

150. The ratio between the total expenditure on Taxes for all the years and the total expenditure on Fuel and
Transport for all the years respectively is approximately?
(a) 4:7 (b) 10:13 (c) 15:18 (d) 5:8

Head Office: 127, Zone II, MP Nagar, Bhopal |+91-7676564400| https://www.toprankers.com Page 38 of 40
Notes:

Head Office: 127, Zone II, MP Nagar, Bhopal |+91-7676564400| https://www.toprankers.com Page 39 of 40
Notes:

Head Office: 127, Zone II, MP Nagar, Bhopal |+91-7676564400| https://www.toprankers.com Page 40 of 40

You might also like